Practice Exam

Pataasin ang iyong marka sa homework at exams ngayon gamit ang Quizwiz!

Which of these is not considered a profit-sharing plan? A) Cash balance plan B) Employee stock option plan (ESOP) C) Stock bonus plan D) Traditional Section 401(k) plan

A) Cash balance plans are not profit-sharing plans (LO 1.2.1). The other plans listed are profit-sharing plans. They don't require fixed annual employer contributions and are not affected by the minimum funding standard requirements. LO 3.1.1

Which of these statements regarding Section 457 plans is CORRECT? A) Earnings on assets in a Section 457 plan grow tax-deferred until withdrawn. B) Deductibility of plan contributions is an important factor for employers choosing a Section 457 plan to consider. C) Required minimum distribution rules do not apply. D) A Section 457 plan is a qualified deferred compensation plan.

A) Earnings on assets in a Section 457 plan grow tax-deferred until withdrawn. Deductibility of plan contributions is not a factor for employers choosing a Section 457 plan because these employers are tax-exempt. Required minimum distribution rules apply. A Section 457 plan is a nonqualified deferred compensation plan. LO 4.3.2

Reed, age 45, has come to you for help in planning his retirement. He works for a manufacturing company, where he earns a salary of $75,000. Reed would like to retire at age 65. He feels this is a realistic goal because he has consistently earned 9% on his investments and inflation has averaged 3%. If Reed expects to live until age 90 and he has a wage replacement ratio of 80%, assuming a capital preservation approach, how much will Reed need to have accumulated on the day that he retires to adequately provide for his retirement lifestyle? A) $1,663,516 B) $1,490,649 C) $1,010,264 D) $1,971,034

A) The answer is $1,663,516. Step 1: Determine the present value of capital needs: Current income$75,000Wage replacement ratio× 80%Present value of capital needs$60,000 Step 2: Determine the future value of the capital needs in the first year of retirement: PV of capital needs($60,000)N (number of years until retirement)20I/YR (use inflation rate)3%FV (required income in the first year of retirement)$108,366.6741 Step 3: Determine the amount of savings (capital) needed at retirement to fund expenses throughout remainder of life expectancy using a capital utilization approach: PMTAD(annuity due) ($108,366.6741) N (retirement life expectancy)25 (90 − 65)I/YR (use real rate of return)5.8252 [(1.09 ÷ 1.03) − 1] × 100PV (capital needed at retirement)$1,490,649 Step 4: For capital preservation, calculate the additional capital needed at retirement to generate the desired retirement income and leave a balance at life expectancy equal to the original capital utilization value: FV = $1,490,649 N = 25 I/YR = 9 Solve PV = (172,867) Add to previous step: $1,490,649 + $172,867 = $1,663,516 LO 8.3.2

Richard participates in a traditional defined benefit pension plan at work. His projected monthly benefit under the plan is $1,000. If the plan provides life insurance for Richard, the death benefit payable under the policy is limited to A) $100,000. B) $17,500. C) $52,000. D) $215,000.

A) The answer is $100,000. Defined benefit plans use the 100 times test for determining whether they comply with the incidental benefit rules. Under this test, the death benefit cannot exceed 100 times the participant's projected monthly benefit (in this instance, $100,000). LO 8.2.2

Sally, age 37, works for two employers, ABC Corporation and XYZ Corporation, both of which maintain Section 401(k) plans. If Sally defers $6,000 to ABC's Section 401(k) plan in 2023, how much can she then defer to XYZ's plan this year? A) $16,500 B) $30,000 C) $66,000 D) $22,500

A) The answer is $16,500. The maximum allowable elective deferral for 2023 is $22,500. If Sally contributes $6,000 to ABC's plan, then she can only contribute up to $16,500 to XYZ's plan ($22,500 − $6,000 = $16,500). LO 4.4.1

John Irving, the 55-year old owner of ABC Corporation, wants to implement a new comparability plan. John's salary is $150,000. The remaining eligible participant census is as follows: AgeSalaryEmployee A35$50,000Employee B33$45,000Employee C54$60,000Employee D41$35,000Employee E43$36,000 If John wants to contribute an aggregate total of $41,300 to the plan this year, what is the maximum amount John can contribute to the comparability plan for himself? A) $30,000 B) $15,000 C) $49,000 D) $37,500

A) The answer is $30,000. A new comparability plan will only satisfy the nondiscrimination rules if the plan design satisfies one of either of these: Each eligible non-highly compensated employee (HCE) must receive an allocation of at least 5% of compensation. If the plan provides for an allocation rate of less than 5%, the minimum allocation rate for the non-HCEs is one-third of the highest allocation rate under the plan. John can contribute a maximum $30,000 (20%) of his salary to the plan while limiting his other employees to as little as 5% of salary. In this example, the total compensation of the eligible employees is $226,000. A 5% contribution for this group totals $11,300, leaving $30,000 of the total $41,300 for John's benefit. LO 3.2.1

Jack is a single taxpayer who retired at age 62 and receives a qualified plan pension of $1,500 each month. He has begun working as a consultant to various firms and is projecting he will earn $70,000 this year. What is the maximum deductible contribution Jack may make to a traditional IRA for 2023? A) $7,500 B) $22,500 C) $0 D) $6,500

A) The answer is $7,500. Jack is not currently an active participant in a qualified plan, is age 50 or older, and has earned income in 2023. Jack may make a deductible IRA contribution of $7,500 ($6,500 + $1,000 catch-up) for 2023. LO 5.1.1

Steve retired from ABC Corporation this year and received a lump-sum distribution from ABC's qualified retirement plan. The distribution consisted entirely of ABC stock valued at $200,000 on the date of distribution. The fair market value of the stock at the time of contribution to the plan was $80,000. Assuming Steve does not sell the stock this year, what amount is included in Steve's gross income this year as a result of the distribution? A) $80,000 B) $0 C) $200,000 D) $120,000

A) The answer is $80,000. Because the distribution is a lump-sum distribution of employer stock, the net unrealized appreciation (NUA) concept applies. Under the NUA rules, the adjusted basis of the stock to the plan trust ($80,000) is included in Steve's gross income in the year of the distribution and is treated as ordinary income. When Steve later sells the stock, he will have an $80,000 basis in it. LO 6.1.1

When is an employee stock ownership plan (ESOP) an appropriate choice for an employer to implement? The employer is either a C or an S corporation. Creating a market for the employer stock helps diversify the employer-owner's financial situation. The employer wishes to increase the company's liquidity by pledging the stock for a loan in the name of the ESOP. The employer wishes to transfer ownership of the business to the employees. A) I, II, III, and IV B) I and IV C) I and III D) II and III

A) The answer is I, II, III, and IV. All these statements regarding ESOPs are correct. An employer may increase the company's liquidity by pledging the stock for a loan in the name of the ESOP. The employer can be either a C or an S corporation. In general, an owner who wants to transfer wealth to family members should not use an ESOP because it transfers wealth to the workers instead of the owner keeping all the wealth from the firm. In other words, an ESOP can dilute current ownership percentages. LO 3.2.2

A qualified plan is A) a tax-efficient way to save for retirement. B) only applicable for firms with 50 or more employees. C) considered a plan that benefits highly compensated employees only. D) a company-sponsored retirement plan with benefits guaranteed by the Employee Retirement Income Security Act (ERISA).

A) The answer is a tax-efficient way to save for retirement. ERISA does not guarantee plan benefits; the Pension Benefit Guaranty Corporation (PBGC) guarantees benefits in defined benefit plans. Qualified plans may be established for firms with as few as one employee. Qualified plans also benefit non-highly compensated employees. (LO 1.3.1). LO 1.2.1

Which of the following regarding Social Security funding is (are) CORRECT? Social Security is funded through a series of taxes paid by the participant and the participant's employer. Social Security is funded through a Federal Insurance Contributions Act (FICA) tax and the self-employment (SECA) tax. A) Both I and II B) Neither I nor II C) I only D) II only

A) The answer is both I and II. Social Security is funded through a series of taxes paid by both the participant and participant's employer, commonly called payroll taxes. A self-employed individual must pay both the employer and employee portions of the FICA tax, known as the self-employment (SECA) tax. LO 7.1.1

Bill's employer maintains a target benefit plan. Bill is age 59. The plan was originally designed to benefit a 38-year-old key employee. There is also substantial turnover at Bill's company. Which of the following statements is CORRECT? A) Contributions to the retirement plan are mandatory. B) Bill's retirement benefit is funded through elective deferrals. C) Bill knows exactly what retirement benefit to expect. D) Forfeitures are likely to be allocated equally to Bill and the 38-year-old key employee.

A) The answer is contributions are mandatory for a target benefit plan. Benefits depend on such plan's account balances, and the final benefit amount is not guaranteed. Target benefit pension plans are funded by the employer, not through employee elective deferrals. Forfeitures in such plans are likely to be unequal as a result of unequal compensation. LO 2.3.2

Basic provisions of SIMPLE IRAs include which of these? A) Employees who earned $5,000 during any two preceding years and are reasonably expected to earn at least $5,000 during the current year must be allowed to participate. B) They are subject to actual deferral percentage test (ADP) nondiscrimination rules. C) Employees might not be fully vested in employer contributions until completing six years of service. D) Long-term employees can benefit from reallocated forfeitures.

A) The answer is employees who earned $5,000 during any two preceding years and are reasonably expected to earn at least $5,000 during the current year must be allowed to participate. Employees are 100% vested in their elective deferrals. SIMPLE IRAs are not subject to the nondiscrimination rules generally applicable to qualified plans (including top-heavy rules). The employee is 100% vested in both his elective deferrals as well as any employer contributions. LO 4.2.1

Which one of these is a correct statement about the constructive receipt doctrine? A) It may tax income that is made available but is not yet received by a taxpayer. B) This doctrine states that when the employee's benefit has become substantially vested or essentially equivalent to the receipt of cash, current income taxation will result. C) It taxes payments made in the future that are based on a company's earnings. D) It prevents a deferred compensation agreement from being informally funded.

A) The answer is it may tax income that is made available but is not yet received by a taxpayer. The constructive receipt doctrine taxes income that is made available, even though the income is not actually received. The constructive receipt doctrine applies to formally funded plans, not unfunded (informally funded) plans. Unfunded plans are based on promise only. The economic benefit doctrine states that when the employee's benefit has become substantially vested or essentially equivalent to the receipt of cash, current income taxation will result. LO 9.1.2

Which one of these is a CORRECT statement about a funded nonqualified deferred compensation plan that funds future benefits and is not currently taxable to plan participants? A) It must be subject to a substantial risk of forfeiture. B) The benefit must be protected by a surety bond. C) The benefit is subject to the claims of the employer's creditors. D) It will provide tax deferral for an employer.

A) The answer is it must be subject to a substantial risk of forfeiture. To avoid immediate taxation to the participant, a funded nonqualified deferred compensation plan must be nontransferable and subject to a substantial risk of forfeiture. LO 9.3.1

Which of these retirement plans generally does not permit in-service withdrawals until age 59½? A) Money purchase pension plans B) Section 401(k) plans C) SEP plans D) Profit-sharing plans

A) The answer is money purchase pension plans. Pension plans (traditional defined benefit, cash balance, target benefit, or money purchase) generally do not allow in-service withdrawals to participants under the age of 59½. All the others listed allow for in-service withdrawals if the plan document permits. LO 6.1.1

ABC Company would like to establish a retirement plan incorporating the following objectives: Attract and retain employees. The employer will make all contributions to the plan with company stock. The plan must integrate with Social Security. What type of retirement plan best suits ABC's objectives? A) Stock bonus plan B) Money purchase pension plan C) Salary reduction SEP (SARSEP) D) Employee stock ownership plan (ESOP)

A) The answer is stock bonus plan. A stock bonus plan would accomplish ABC Company's objectives. The others are not the best plan for these reasons: Money purchase pension plans only allow for 10% company stock. ESOPs cannot be integrated with Social Security. New SARSEPs can no longer be established. LO 8.1.2

What is the taxable character of distributions that are made from a Roth IRA? A) Tax-free income if the distribution meets the holding period and qualified distribution requirements B) Tax-deferred income when converted to a traditional IRA C) Capital gain income if the distribution meets the required holding period D) Ordinary income if the taxpayer fails to make required minimum distributions

A) The answer is tax-free income if the distribution meets the holding period and qualified distribution requirements. Distributions made from a Roth IRA are income tax free if the Roth IRA meets certain specified conditions. These conditions include meeting the 5-year holding period and 1 of the following: (1) the owner is age 59½ or older, (2) distribution upon disability of the owner, (3) distribution to a beneficiary upon the death of the owner, or (4) for a first-time home purchase (up to a $10,000 lifetime cap). LO 5.4.1

Under a profit-sharing plan, A) the company has flexibility as to annual funding. B) the employer bears investment risk. C) up to 25% of the plan's assets can be invested in the employer's stock. D) the company must make annual contributions.

A) The answer is the company has flexibility as to annual funding. Pension plans can invest up to 10% only of plan assets in employer stock. Profit-sharing plans have no restrictions regarding investment in employer stock. The employer may deduct a contribution limited to only 25% of participating employees' covered compensation. The employer must make substantial and recurring employer contributions, or the IRS will remove the plan's qualified status. The employee bears investment risk. LO 3.3.1

Jill has decided to offer a retirement plan to her employees. She wants to implement a savings incentive match plan for employees (SIMPLE) and is trying to decide between a SIMPLE IRA and a SIMPLE 401(k). All of these statements apply to both types of SIMPLE plans except A) there is a 25% penalty for early distributions from a participant's account within two years of entry into the plan. B) SIMPLEs are not subject to the top-heavy rules that apply to qualified plans. C) employer contributions are not subject to payroll taxes (FICA and FUTA). D) Either type of SIMPLE may accept Roth contributions.

A) The answer is there is a 25% penalty for early distributions from a participant's account within two years of entry into the plan. Only early distributions from a SIMPLE IRA within the first two years of initial participation in the plan are subject to the 25% early withdrawal penalty tax. LO 3.3.3

Regarding assumptions used in retirement needs analysis calculations, which of these is (are) CORRECT? All other things being equal, increasing the life expectancy of the retiree will lower the amount of capital needed on the first day of retirement to support the assumed retirement income. All other things being equal, changing the assumed rate of return from 6% to 8% and the assumed inflation rate from 2% to 4%, will lower the amount of capital needed on the first day of retirement to support the assumed retirement income. A) Neither I nor II B) I only C) Both I and II D) II only

A0 The answer is neither I nor II. Increasing the life expectancy of the retiree increases the amount of capital needed on the first day of retirement to support the assumed retirement income because the retiree will draw on the capital fund for a longer period. Statement II is incorrect because an 8% investment return and 4% inflation rate produces a lower inflation-adjusted rate of return, and thus increases the amount of capital needed on the first day of retirement to support the assumed retirement income. [(1.06 ÷ 1.02) − 1] × 100 = 3.9216%; [(1.08 ÷ 1.04) − 1] × 100 = 3.8462% LO 8.3.1

(Case Study Question) Chris's employer, Gilmore Glass Company, is considering making an additional profit-sharing contribution to the company's Section 401(k) plan for this year. What is the maximum amount of additional profit-sharing contribution the company could make on behalf of Chris for 2023? A) $22,500 B) $61,800 C) $18,300 D) $66,000

B) The answer is $61,800. Chris contributes 3% of his salary ($70,000 × 3% = $2,100) and the employer matches employee deferrals up to that amount. The maximum that can be contributed to Chris's profit-sharing account is the lesser of 100% of his compensation or $66,000. Because $2,100 is going in as an elective deferral from Chris's salary and another $2,100 is the match, that leaves $61,800 the employer could contribute. LO 10.1.2

Blake, age 73, is required to take substantial required minimum distributions (RMDs) from his qualified retirement plan. He has no current need for the income and wants to decrease the amount of the distributions without incurring a penalty. Blake is not interested in totally withdrawing all assets from the plan at this time. Which of the following statements regarding Blake's options is CORRECT? Blake may take a distribution in addition to his RMD from his qualified plan and convert the additional distribution to a Roth IRA within 60 days. Blake cannot roll over retirement plan proceeds to a traditional IRA after age 73. A) Both I and II B) I only C) II only D) Neither I nor II

B) The answer is I only. If Blake takes a distribution that is in addition to his RMD, he can pay the required income tax on this distribution and convert it to a Roth IRA. Because there are no required minimum distributions for Roth IRAs, Blake will have effectively reduced the amount of his pretax retirement plan account against which he must calculate subsequent RMDs. Statement II is incorrect. A direct transfer or rollover may occur at any age. LO 6.2.1

Which of the following statements regarding the tax effects of converting a traditional IRA to a Roth IRA is (are) CORRECT? The converted amount is treated as a taxable distribution from the traditional IRA. The 10% premature penalty applies if the owner is not at least 59½ years old. A) Both I and II B) I only C) II only D) Neither I nor II

B) The answer is I only. When a traditional IRA is converted to a Roth IRA, the converted amount is treated as a taxable distribution and is included in the owner's gross income. The 10% penalty does not apply to the conversion amount when converted, regardless of the owner's age. However, if the taxable portion of the converted amount is withdrawn within five years of the conversion, then the taxable portion of the conversion is treated as coming out first when the converted amount is withdrawn. This taxable amount would be subject to the early withdrawal rules and penalized 10% unless an exception applies. The point of this rule is to protect Roth conversions from being sham transactions intended to get around the 10% early withdrawal penalty. The law treats withdrawals of converted amounts that are more than five years past the conversion date the same as contributions. LO 5.4.2

A business owner-client approaches a financial planner for advice on selecting a retirement plan for the business. What factors should guide the financial planner's recommendations? The owner's retirement savings need The owner's current age The amount of risk the client is comfortable assuming The financial stability of the business. A) II and III B) I, II, III, and IV C) III and IV D) I, II, and III

B) The answer is I, II, III, and IV. All the factors listed should be considered in selecting a retirement plan for the business. LO 8.1.1

In a money purchase pension plan, forfeitures revert to the plan. may be used to reduce future employer contributions. can be reallocated among the remaining plan participants. do not count against remaining participants' annual additions limits. A) I, II, III, and IV B) I, II, and III C) I and II D) III and IV

B) The answer is I, II, and III. Forfeitures count against the remaining participants' annual additions limits. Forfeitures may be used to reduce future employer contributions. They revert to the plan and they count against remaining participants' annual additions limits. LO 2.3.1

Which of the following are minimum coverage tests for qualified retirement plans? Nondiscrimination test Average benefits percentage test Ratio test Maximum compensation test A) II, III and IV B) II and III C) I and II D) I, II and III

B) The answer is II and III. The two minimum coverage tests for qualified retirement plans are the average benefits percentage test and the ratio test. In order to be qualified, a retirement plan must meet at least one of these tests if the plan does not meet the percentage (safe harbor) test. LO 1.3.1

Which of these is an example of a tax-advantaged plan? A) Cash balance plan B) Section 403(b) plan C) Section 401(k) plan D) Employee stock ownership plan (ESOP)

B) The answer is Section 403(b) plan. A Section 403(b) plan is a tax-advantaged plan but not an ERISA-qualified retirement plan. While tax-advantaged plans are very similar to qualified plans, there are some minor differences. For example, a tax-advantaged plan is not allowed to have net unrealized appreciation (NUA) treatment. They are also not allowed to offer 10-year forward averaging or special pre-1974 capital gains treatment. Tax-advantaged plans also have less restrictive nondiscrimination rules; otherwise they are very similar to qualified plans. Cash balance plan, Section 401(k) plan, and Employee stock ownership plan (ESOP) are examples of qualified retirement plans. LO 1.2.1

The premature distribution penalty does NOT apply to which of these IRA distributions? A) A distribution due to total and permanent disability B) A distribution made after the owner is age 55 and after separation from service C) A distribution paid to a beneficiary after the death of the IRA owner who had not begun receiving minimum distributions D) A distribution made for purpose of paying qualified higher education costs

B) The answer is a distribution made after the owner is age 55 and after separation from service is only an exception for distributions from qualified plans, not from IRAs. LO 5.3.1

Which of the following qualified plan distributions is subject to the 10% penalty for early withdrawal? A) A lump-sum distribution made to an employee-participant, age 63, from a profit-sharing plan after the funds have been in the plan for two years B) An in-service hardship distribution from a Section 401(k) plan to an employee-participant, age 55 C) A death benefit payable to a beneficiary upon the death of an employee, age 52 D) A lump-sum benefit payable to a disabled employee-participant, age 57

B) The answer is an in-service hardship distribution from a Section 401(k) plan to an employee-participant, age 55. Even if the distribution is a hardship withdrawal, the penalty applies unless the employee-participant has attained the age of 59½ (or one of the other 10% penalty exceptions applies). LO 6.3.1

(Case Study Question) If you are unable to obtain sufficient and relevant quantitative information and documents from the Bernards to form a basis for recommendations, which of the following could you, a CFP® professional, do to follow the Code of Ethics and Standards of Conduct? Terminate the engagement. Restrict the scope of the engagement. A) Neither I nor II B) Both I and II C) II only D) I only

B) The answer is both I and II. According to the Code and Standards, "if the practitioner is unable to obtain sufficient and relevant quantitative information and documents to form a basis for recommendations, the practitioner shall either restrict the scope of the engagement to those matters for which sufficient information is available; or terminate the engagement." LO 10.3.2

Which of the following statements regarding plan forfeitures in a money purchase pension plan is(are) CORRECT? Plan forfeitures may be used to reduce future employer contributions. An employer may reallocate the plan forfeitures among the remaining plan participants, increasing their potential individual account balances but only up to the annual additions limit for each participant. A) I only B) Both I and II C) Neither I nor II D) II only

B) The answer is both I and II. Both of these statements are correct for money purchase pension plans. Forfeitures may be used to reduce future employer contributions. They revert to the plan and count against remaining participants' annual additions limits. LO 2.3.1

Which statement does not correctly describe a concept related to nonqualified deferred compensation or stock plans? A) In a salary reduction plan, the employee elects to give up a specified portion of current compensation. B) A disqualifying disposition with an incentive stock option (ISO) will result in ordinary income taxes, as well as FICA and FUTA taxes. C) If an employee makes a Section 83 election upon the grant of restricted stock, he or she will not be taxed when the restricted stock becomes vested. D) Employee stock purchase plans (ESPPs) can be offered at a discount, unlike ISOs.

B) The answer is disqualifying disposition with an ISO will result in ordinary income taxes, as well as FICA and FUTA taxes. ISOs, when there is a disqualifying disposition, are subject to ordinary taxes, but they are not subject to FICA or FUTA taxes. LO 9.4.1

A preretirement distribution from a qualified retirement plan can escape the 10% early withdrawal penalty in each of the following situations EXCEPT A) distributions made as part of a series of substantially equal periodic payments made at least annually over the life or life expectancy of the employee or the joint lives or life expectancies of the employee and beneficiary. B) distributions made after a separation from service at any age. C) distributions made to cover medical expenses in excess of 7.5% of adjusted gross income for a taxpayer who is age 55. D) distributions made to a beneficiary or to an employee's estate on or after the employee's death.

B) The answer is distributions made after a separation from service for early retirement after any age. For a preretirement distribution to escape the 10% penalty for early distribution, the distribution must be made after a separation from service in the year the worker turns age 55 or later. This exception is not applicable to IRAs. LO 6.1.1

Napoleon Enterprises sponsors a SIMPLE 401(k) for its employees. Under the plan, the company matches employee contributions up to 3% of compensation. Which of these statements about Napoleon Enterprises' SIMPLE 401(k) is CORRECT? A) Napoleon Enterprises' contributions must be vested using either a 3-year cliff or 2-to-6-year vesting schedule. B) Employees can make Roth contributions to the plan. C) Withdrawals made within two years of initial participation are subject to a 25% premature distribution penalty tax. D) Napoleon Enterprises can match as little as 1% of compensation for two out of five years.

B) The answer is employees can make Roth contributions to any SIMPLE plan. This is new starting in 2023 under SECURE 2.0. Employee after-tax contributions are not allowed. All the other statements are incorrect: Unlike SIMPLE IRAs, employers that sponsor SIMPLE 401(k)s cannot reduce the matching percentage to below 3%. Employer contributions to a SIMPLE 401(k) are 100% vested (LO 4.2.1). The 25% penalty applies only to SIMPLE IRAs. LO 3.3.3

Several years ago, Greener Grass Company implemented a traditional defined benefit plan. According to the plan document, the employer must contribute an annual amount that will provide the employees with a specified benefit at retirement. Which of these events would be expected to decrease the employer's annual contribution to a traditional defined benefit pension plan using a percentage for each year of service benefit formula? A) Benefits are cost of living adjusted. B) investment returns of the plan are greater than expected. Forfeitures are less than anticipated would result in increased contributions. C) Inflation is higher than expected. D) Forfeitures are less than anticipated.

B) The answer is investment returns of the plan are greater than expected. Forfeitures are less than anticipated would result in increased contributions. Inflation would likely cause salaries and plan expenses to increase, thereby causing contributions to increase. Likewise, benefits that are adjusted for the cost of living would result in greater employer contributions, not less. LO 1.3.2

Which of the following is one of the differences between defined benefit pension plans and defined contribution plans? A) A defined contribution plan has a benefit limit, whereas a defined benefit pension plan has a contribution limit. B) Investment risk is borne by the employer in a defined benefit pension plan, whereas the employee bears the risk in a defined contribution plan. C) A guaranteed retirement benefit is the goal of a defined contribution plan, while a guaranteed contribution is the focus of a defined benefit pension plan. D) Accounts are commingled in a defined contribution plan, while a defined benefit pension plan maintains separate accounts for each participant.

B) The answer is investment risk is borne by the employer in a defined benefit pension plan, whereas the employee bears the risk in a defined contribution plan. A defined benefit pension plan has a benefit limit, whereas a defined contribution plan has a contribution limit. Investment risk is borne by the employer in a defined benefit pension plan, whereas the employee bears the risk in a defined contribution plan. Accounts are commingled in a defined benefit pension plan, and a defined contribution plan maintains separate accounts for each participant. A guaranteed retirement benefit is the goal of a defined benefit pension plan, while a guaranteed contribution is the focus of a defined contribution plan. LO 2.1.1

Which of these statements regarding Roth IRAs and pre-tax 401(k) plans is (are) CORRECT? Roth IRAs require distributions no later than age 73 while the participant is living. There is not an income limitation to participate in a pre-tax 401(k) plan or Roth IRA. A) II only B) Neither I nor II C) I only D) Both I and II

B) The answer is neither I nor II. Although there is not an income limitation to participate in a pre-tax 401(k) plan, there are income limits for Roth IRAs (2023 - modified AGI: $228,000 married/$153,000 single). For Roth IRAs, there is no requirement to start taking distributions while the participant is living. For pre-tax 401(k) plans, distributions must begin no later than age 73, unless the participant is still working at the company and is not a 5% owner. LO 5.4.3

Robert established a Roth IRA. He turns age 73 this year. Which of the following statements is (are) CORRECT? Robert must begin taking required minimum distributions (RMDs) by April 1 of next year. Robert can no longer make contributions to the Roth IRA. A) Both I and II B) Neither I nor II C) I only D) II only

B) The answer is neither I nor II. The original owner of a Roth IRA is never subject to an RMD requirement during his lifetime and can continue making contributions after he reaches age 73. LO 5.4.1

Which of these statements regarding the coverage rules for qualified plans is FALSE? A) The coverage tests for qualified plans include the ratio test. B) The coverage tests for qualified plans include the average contribution percentage test. C) The coverage tests for qualified plans include the average benefits percentage test. D) A retirement plan can cover any portion of the workforce, provided it satisfies one of the three coverage tests under Section 410(b).

B) The answer is the coverage tests for qualified plans include the average contribution percentage test is false. The third coverage test is the average benefit percentage test, not the average contribution percentage test. LO 1.3.1

If an executive has unrestricted access to the funds set aside for her nonqualified deferred compensation (NQDC) plan, A) there is a substantial risk of forfeiture. B) the executive is in constructive receipt of the funds. C) ownership of the funds is retained solely by the employer. D) the plan becomes a qualified plan.

B) The answer is the executive is in constructive receipt of the funds. If the employee has an unrestricted option to take the funds, the employee will be considered to be in constructive receipt of the income and will be taxed accordingly in the current year. LO 9.3.2

Which statement regarding integrating a plan with Social Security is true? A) Only the excess method can be used by a defined benefit pension plan. B) Under the offset method, a fixed or formula amount approximates the existence of Social Security benefits and reduces the plan formula. C) The maximum increase in benefits for earnings above the covered compensation level is 50% for a defined benefit pension plan. D) Because there is a disparity in the Social Security system, all retirement plans are allowed integration with Social Security.

B) The answer is under the offset method, a fixed or formula amount approximates the existence of Social Security benefits and reduces the plan formula. The maximum increase in benefits for earnings above the covered compensation level is 26.25% for a defined benefit pension plan, not 50%. The excess method may only be used by a defined contribution plan. A defined benefit plan may use either the excess method or the offset method for Social Security integration. Only certain retirement plans may be integrated with Social Security. LO 1.3.3

If a covered worker were to become disabled for Social Security benefit purposes, which individual would be ineligible to also receive a benefit from Social Security based on the disabled worker's record? A) The spouse of the disabled worker B) The worker's child who became disabled at age 24 C) The disabled worker D) An unmarried dependent child of the disabled worker, under age 19 and still in high school

B) The worker's child who became disabled at age 24 would not be eligible for Social Security benefits based on the worker's earnings history when their parent became disabled. The child must have been disabled before attaining age 22 to get Social Security benefits based on a parent's work history. LO 7.2.1

While Section 403(b) (tax-sheltered annuity plan or TSA) plans are an excellent source of retirement savings, they do have some disadvantages, such as A) actual deferral percentage (ADP) testing causes Section 403(b)/TSA plans to be relatively costly and complex to administer. B) account balances at retirement age are guaranteed to be sufficient to provide adequate retirement amounts for employees who entered the plan at later ages. C) Section 403(b) plans must comply with the actual contribution percentage (ACP) test for employer matching contributions. D) investments are limited to mutual funds, individual stocks, and annuities.

C) Section 403(b) plans must comply with the actual contribution percentage (ACP) test for employer matching contributions. Section 403(b)/TSA plan investments are limited to mutual funds and annuities. Although the ADP test does not apply, Section 403(b)/TSA plans must comply with the ACP test for matching contributions. One way to remember that Section 403(b) plans must pass ACP testing and not ADP testing is that Section 403(b) plans are for 501(c)(3) organizations. The (c) in 501(c)(3) is like the "C" in ACP testing. Nondiscrimination testing causes Section 403(b)/TSA plans to be relatively costly and complex to administer. Account balances at retirement age may not be sufficient to provide adequate retirement amounts for employees who entered the plan at later ages. Section 403(b)/TSA plan investments are limited to mutual funds and annuities. LO 4.3.1

(Case Study Question) Assuming Chris Bernard's income increases by 3% per year between now and age 62, what lump sum will be necessary at the date of retirement to provide Chris's goal of 80% of preretirement income for 25 years in retirement without considering Social Security? Assume the Bernards' required rate of return in the calculation. A) $2,313,975 B) $1,400,000 C) $1,388,038 D) $839,790

C) The answer is $1,388,038. The lump sum needed at the date of retirement to support Chris's goal (without considering Social Security) is $1,388,038. Step 1: $70,000 × 80% = $56,000 $56,000, +/−, PV 17, N 3, I/YR Solve FV $92,559 Step 2: BEG mode $92,559, PMT 25, N [(1.08 ÷ 1.03) − 1 × 100] = 4.8544, I/YR 0, FV Solve PV −$1,388,038 LO 10.2.1

Reed, age 45, has come to you for help in planning his retirement. He works for a manufacturing company, where he earns a salary of $75,000. Reed would like to retire at age 65. He feels this is a realistic goal because he has consistently earned 9% on his investments and inflation has only averaged 3%. Assuming he is expected to live until age 90 and he has a wage replacement ratio of 80%, how much will Reed need to have accumulated on the day that he retires to adequately provide for his retirement lifestyle? A) $1,408,595 B) $1,863,311 C) $1,490,649 D) $1,160,240

C) The answer is $1,490,649. Step 1: Determine the present value of capital needs: Current income$75,000Wage replacement ratio× 80%Present value of capital needs$60,000 Step 2: Determine the future value of the capital needs in the first year of retirement: PV of capital needs($60,000)N (number of years until retirement)20I/YR (use inflation rate)3FV (required income in the first year of retirement)$108,366.6741 Step 3: Determine the amount of savings (capital) needed at retirement to fund expenses throughout remainder of life expectancy: PMTAD(annuity due - Beg Mode)($108,366.6741)N (retirement life expectancy)25 (90 − 65)I/YR (use real rate of return)5.8252 [(1.09 ÷ 1.03) − 1] × 100PV (capital needed at retirement)$1,490,649 LO 8.3.2

Ross Company has a traditional Section 401(k) plan. The actual deferral percentage (ADP) for all eligible non-highly compensated employees (non-HCEs) is 4%. What is the maximum ADP for the highly compensated employees (HCEs) group at Ross Company? A) 5.25% B) 5.7% C) 6% D) 4%

C) The answer is 6%. The maximum ADP for HCEs at the Ross Company is 6%. To satisfy the ADP test, a traditional 401(k) plan must meet one of the following two tests. The ADP for eligible HCEs must not be more than the ADP of all other eligible employees multiplied by 1.25. In this case, the non-HCEs averaged 4%. (4% × 1.25 = 5%) The ADP for eligible HCEs must not exceed the ADP for other eligible employees by more than 2% (4% + 2% = 6%), and the ADP for eligible HCEs must not be more than the ADP of all other eligible employees multiplied by 2 (4% × 2 = 8%). This second test is a lesser-than test. The lesser number is 6%. Thus, 6% is the second test answer. Thus, the two test answers are 5% and 6%. The plan only must pass one of these tests, and the HCEs want to know how much they can contribute, so the higher number always wins. That is 6% in this case. LO 3.3.2

Martha has been impressed with the appreciation of the coin collection she received as a gift from her mother and would like to take advantage of this by using coins as an investment in the IRAs. Which of these statements regarding coins as investments in IRAs is CORRECT? A) Any government-issued gold coins, such as Krugerrands and American Eagles, are appropriate for IRA investment. B) Martha should approach her coin dealer and ask that a collection similar to the collection her mother gave her be created as an investment for the IRAs. C) American Eagle gold coins are permitted IRA assets. D) No more than 25% of Martha's IRA assets may be invested in coins.

C) The answer is American Eagle gold coins are permitted IRA assets. Only permissible collectible that an IRA may invest in is certain U.S. coins, such as the American Eagle gold coin. LO 8.2.1

Jack inherited his father's Section 401(k) plan account. His father passed away at age 70. Which of these statements regarding Jack's options is (are) CORRECT? Jack is permitted to use a direct trustee-to-trustee transfer of the plan balance into an inherited IRA account. If Jack creates an inherited IRA with the benefit, he can designate his own beneficiary to the account. The payout of the benefit in an inherited IRA is over the lifetime of the original account holder, Jack's father. A) II and III B) I only C) I and II D) I, II, and III

C) The answer is I and II. Statement III is incorrect. Jack is a designated beneficiary (not an eligible designated beneficiary). Thus he is under the ten-year rule for when the decedent passed before their required beginning date (RBD). LO 5.2.1

Which of these are true of the actual contribution percentage test (ACP) test for 401(k) plans? The ACP test is not used unless a 401(k) plan has a match or allows employee after-tax contributions. The ACP test uses the same two test structure and percentage rules as the ADP test. The ADP test accounts only for employee deferrals. The ACP test accounts for employer matching and after-tax contributions, but not pretax contributions and elective deferrals. If the ADP of the non-highly compensated employees is greater than 2% but less than or equal to 8%, then the maximum ADP of the highly compensated employees is 2% more than the ADP of the non-highly compensated employees. A) I, II, and IV B) I, III, and IV C) I, II, III, and IV D) II and III

C) The answer is I, II, III, and IV. All statements are true. LO 3.3.2

Which of the following statements regarding fully insured Section 412(e)(3) plans is(are) CORRECT? A fully insured plan is inappropriate for an employer who cannot commit to regular premium payments. This type of plan is not required to be certified by an enrolled or licensed actuary. All Section 412(e)(3) plans must meet minimum funding standards each plan year. A Section 412(e)(3) plan is a type of defined benefit pension plan. A) II and IV B) I and II C) I, II, and IV D) I, II, III, and IV

C) The answer is I, II, and IV. Statement III is incorrect. Section 412(e)(3) plans must only meet minimum funding standards if there is a loan outstanding against the insurance policy funding the plan. LO 2.2.2

Adam, age 48, and Mary, age 47, were married for 15 years when they divorced last year. Adam died this year. They have two young children, ages 10 and 12, who are cared for by Mary. Adam's 70-year-old mother, Sarah, also survived him. At the time of Adam's death, he was currently, but not fully, insured under Social Security. What benefits are Adam's survivors entitled to under the Social Security program? A dependent parent's benefit A lump-sum death benefit of $255 A children's benefit based on a percentage of Adam's primary insurance amount (PIA) A surviving spouse benefit to take care of a dependent child A) I and II B) I, II, and III C) II, III, and IV D) III and IV

C) The answer is II, III, and IV. One lump-sum death benefit of $255 is payable if he was fully or currently insured. The children's benefit is payable because Adam was currently insured. Adam's divorced spouse (the children's mother) is entitled to a caretaker's benefit for caring for his children under age 16. Adam's mother would only be entitled to a benefit if Adam was fully insured and he had been providing at least half of her support at the time of his death. LO 7.2.2

Why would a qualified retirement plan include real estate among its portfolio of investment assets? A) Because real estate is always considered to be a very low risk holding B) To provide increased liquidity C) As an inflation hedge D) To fund future fixed obligations

C) The answer is as an inflation hedge. The inclusion of real estate among a retirement plan's investment portfolio is most suitable as a hedge against future inflation. Otherwise, it is generally nonliquid, is not stable enough to fund fixed obligations, and is not always considered a very low risk holding. LO 8.2.1

A traditional IRA is appropriate when it is considered an important supplement or alternative to a qualified pension or profit-sharing plan. sheltering current compensation or earned income from taxation is a taxpayer's goal. A) II only B) Neither I nor II C) Both I and II D) I only

C) The answer is both I and II. A traditional IRA is also suitable when a taxpayer wishes to accumulate assets for retirement or defer taxes on investment income. LO 5.1.1

To qualify for disability income benefits under Social Security, a worker must have an impairment that A) is related solely to alcoholism. B) is related solely to drug addiction. C) is expected to last at least 12 months or result in death. D) is expected to result in death within 6 months.

C) The answer is expected to last at least 12 months or result in death. To qualify for Social Security disability income benefits, a person must suffer an impairment that is expected to last at least 12 months or result in death. The disability also must have lasted at least 5 months before Social Security disability benefits can be paid. LO 7.2.1

Over the past 20 years, Green Corporation stock has appreciated an average of 15% per year. The management of the closely held corporation would like to provide a benefit to certain employees that will allow the employees to take advantage of this growth. However, they do not want to give the employees any control of the company. Analyze the scenario and determine the most appropriate arrangement for Green Corporation. A) Employee stock purchase plan (ESPP) B) Nonqualified stock options C) Phantom stock plan D) Restricted stock plan

C) The answer is phantom stock plan. A phantom stock plan is most appropriate because it credits an employee's account with units that mirror the stock's value and pays the eventual benefit in cash. All of the other choices would result in the employee receiving stock (and thus some control) of the company. LO 9.5.1

Which of these statements regarding prohibited transactions is false? A) One category of prohibited transactions bars a fiduciary from causing the plan to engage in a transaction if the fiduciary knows or should know that such a transaction constitutes a direct or indirect involvement between the plan and the parties in interest. B) One category of prohibited transactions involves self-dealing. C) The lending of money or other extension of credit between the plan and a party in interest (outside of nondiscriminatory retirement plan loans based on the participant's account balance) is not a prohibited transaction. D) One category of prohibited transactions involves the investment in the sponsoring employer's stock or real property above certain limits.

C) The answer is the lending of money or other extension of credit between the plan and a party in interest (outside of nondiscriminatory retirement plan loans based on the participant's account balance) is NOT a prohibited transaction is just plain wrong (false). The sale, exchange, lending, or leasing of any property between the plan's assets and a party in interest is a prohibited transaction. A plan participant can take a retirement plan loan based on the participant's balance, but the plan cannot make a loan of the general plan assets to a party in interest. LO 1.4.1

Total annual contributions to an individual participant in a traditional Section 401(k) plan are limited in 2023 to A) $330,000. B) $15,500. C) the lesser of 100% of compensation, or $66,000. D) $22,500.

C) The answer is the lesser of 100% of compensation, or $66,000. Total annual contributions to a participant's account are limited to the lesser of 100% of employee compensation, or $66,000 (2023), with only the first $330,000 (2023) of employee compensation considered in the contribution formula. The total contribution is made up of the worker contribution, the employer contribution, and reallocated forfeitures. The worker contribution alone is limited to $22,500 in 2023 for those 49 and younger. LO 3.3.1

Which of these statements regarding profit-sharing plans is CORRECT? A) Company profits are required to make contributions to a profit-sharing plan. B) Companies adopting a profit-sharing plan are required to make annual contributions to the plan. C) The maximum tax-deductible employer contribution to a profit-sharing plan is 25% of total includible employee compensation. D) Profit-sharing plans are only suited for companies with predictable cash flows.

C) The answer is the maximum tax-deductible employer contribution to a profit-sharing plan is 25% of total includible employee compensation. Profits are not required to be able to make a contribution to a profit-sharing plan. A current contribution may be made from retained earnings or current cash flow. Annual contributions are not required in a profit-sharing plan. Profit-sharing plans are suitable for companies with unstable earnings given that they have discretion over contributions. LO 3.1.1

(Case Study Question) A local competitor company has been aggressively recruiting Chris. If Chris leaves Gilmore Glass, which statement regarding the tax status of any outstanding loan from his Section 401(k) plan is CORRECT if Chris cannot arrange to pay off the loan? A) The loan balance continues to be tax free because Chris is essentially borrowing from himself. B) The outstanding loan is not subject to income tax if not repaid. C) the outstanding loan may be subject to an additional 10% excise penalty. D) Chris may roll over the outstanding loan to an IRA and continue paying the loan under the same rules as before.

C) The outstanding loan may be subject to an additional 10% excise penalty. No one can roll a retirement plan loan to an IRA and keep the payments and term the same. By using a plan loan offset, Chris could lower the amount of the defaulted loan by making payments into his new employer's plan or an IRA up to the due date (including extensions) of his tax return for the year of the retirement plan loan default. The loan becomes fully taxable if not repaid and may be subject to an additional 10% excise penalty. Since he would be separating from service, the defaulted loan amount would be subject to the QPLO rules. LO 10.2.1

Thad and Debra, both age 48, are married and will file a joint return. Their 2023 modified adjusted gross income is $120,000 (including Thad's $95,000 salary). Debra had no earned income of her own. Neither spouse was covered by an employer-sponsored retirement plan. What is the total maximum deductible contribution Thad and Debra may make to a traditional IRA this year? A) $5,200 B) $6,500 C) $0 D) $13,000

D) The answer is $13,000. Because neither Thad nor Debra participates in an employer-sponsored retirement plan, they can contribute and deduct $6,500 each for 2023. While Debra has no earned income, a spousal IRA may be established and funded based on Thad's compensation. LO 5.1.2

(Case Study Question) Chris and Allison Bernard are dedicated to reducing living expenses to be able to save more for retirement. The couple is not counting any possible income from any potential part-time earnings for Allison because that income is not yet realized. Assuming they create the necessary discretionary income, what is the total maximum deductible contribution they can make to a traditional IRA together for 2023? A) $0 B) $9,000 C) $6,500 D) $13,000

D) The answer is $13,000. The maximum deductible contribution for the Bernards to a traditional IRA for 2023 is $13,000 ($6,500 each). Chris is an active participant in a qualified plan, but his AGI is below the married filing jointly (MFJ) phaseout of $116,000. If Allison has no earned income in 2023, she can make an IRA contribution based on Chris's earned income, and her AGI for these purposes is below the nonparticipant spouse AGI limit of $218,000. LO 10.1.1

(Case Study Question) If Chris died today, what is the total lump-sum death benefit that would be paid by Social Security to his family? A) $1,275 B) $510 C) $1,020 D) $255

D) The answer is $255. It is the total amount paid, regardless of the number of Social Security beneficiaries. LO 10.1.1

Required minimum distributions from a traditional IRA must begin no later than A) April 1 of the year in which the IRA owner attains age 73. B) age 59½. C) December 31 of the year following the IRA owner's attainment of age 73. D) April 1 of the year following the year in which the IRA owner attains age 73.

D) The answer is April 1 of the year following the year in which the IRA owner attains age 73. The Internal Revenue Code provides that minimum distributions from a traditional IRA must begin no later than April 1 of the year following the year in which the IRA owner attains age 73 according to SECURE 2.0. LO 6.2.2

Which of the following statements describing how qualified pension plans differ from SEP and SIMPLE plans is(are) CORRECT? Qualified plan rules provide greater flexibility in the number and makeup of the employees covered by the plan than do the rules pertaining to SEP and SIMPLE plans. Participants must be fully and immediately vested in the contributions to qualified plans, but SEP and SIMPLE plans are permitted to have vesting schedules. A) Neither I nor II B) Both I and II C) II only D) I only

D) The answer is I only. Statement II is incorrect because participants must be fully and immediately vested in the contributions to SEPs and SIMPLEs. Qualified plans can include vesting schedules. LO 4.2.2

Section 401(k) plans must have automatic survivor benefits (QJSAs and QPSAs) unless the plan provides that, upon the participant's death, the vested account balance will be paid in full to the surviving spouse. the plan is not a direct or indirect transferee of a plan to which the automatic survivor annuity requirements apply. the participant elects to receive payment as a lump-sum distribution. the participant does not elect payments in the form of a life annuity. A) III and IV B) I, II, and III C) I and II D) I, II, III, and IV

D) The answer is I, II, III, and IV. For a participant to have elected to receive a lump-sum distribution, the spouse must sign a spousal consent form in front of a notary or an authorized plan representative. LO 6.4.2

Henry works for a firm that offers a 401(k) plan. He makes $75,000/year but he has been hesitant to contribute because he felt he may need the money before retirement. Recently Henry learned that he could receive a loan from his 401(k) plan without paying any income tax. He is now considering putting money into the 401(k), but he wants to know more about loan provisions. Which of the following statements regarding loans from qualified plans are CORRECT? The limit on loans is generally one-half of the participant's vested account balance not to exceed $50,000. The limit on the term of a retirement plan loan is generally five years. If an employee leaves the company, a retirement plan loan may be rolled over to an IRA and the participant may continue making the loan payments as planned. Loans to a 100% owner-employee are permissible. A) I, III, and IV B) I and III C) II and IV D) I, II, and IV

D) The answer is I, II, and IV. Statement I is correct. Generally, loans are limited to one-half the vested account balance and cannot exceed $50,000. Note: When account balances are less than $20,000, however, loans up to $10,000 may be available. Also, for vested balances below $10,000, the entire vested balance may be available for loans. Statement II is correct. The limit on the term of any loan is generally five years, unless the loan is for a principal residence. Loans for the purpose of buying a residence must be repaid over a reasonable period. The rules have also been modified at times in the past due to certain federally declared disasters. Statement III is incorrect. A qualified plan loan may not be rolled over to an IRA, but it would be allowed to be moved to a new employer's retirement plan if the plan document allowed it. An outstanding retirement loan balance that is defaulted is treated as a distribution and thus is subject to income taxes and the early withdrawal penalty rules. However, if the loan default is solely due to separation from service or the retirement plan with the loan is terminated, then plan participants have until the due date of their tax return for the year in which the default occurred (including extensions) to return as much of the money as they can to their IRAs or their new employer's plan (if the plan permits). The amount returned is called a qualified plan loan offset (QPLO) and is treated like a transfer. Statement IV is correct. Loans from qualified plans to sole proprietors, partners, shareholders in S corporations and C corporations are permitted. What is not permitted is a retirement plan making a fixed income investment of general plan assets to a party at interest. LO 6.3.2

Build Corporation communicated several goals to its financial adviser when it decided to implement a qualified retirement plan for the company. After reviewing these goals, the adviser recommended that Build Corporation implement a defined benefit pension plan. Which of the following statements regarding employers who are candidates for a defined benefit pension plan are CORRECT? Typically, they have the objective of instituting a plan with highly predictable costs. They indicate that the desire to provide a tax shelter for key employees outweighs the need for an administratively convenient plan. They want benefit levels guaranteed. They want a simple and inexpensive plan. A) I, II, III, and IV B) I and IV C) I, II, and III D) II and III

D) The answer is II and III. Employers who sponsor defined benefit pension plans typically do not have highly predictable costs, because funding is subject to an annual actuarial determination. Highly paid employees receive a tax shelter with a defined benefit plan in the sense that their compensation includes large contributions to the defined benefit and they are not subject to current income tax on this compensation. The Pension Benefit Guaranty Corporation (PBGC) and the employer guarantee benefit levels. Defined benefit pension plans are complex to design and expensive to install and maintain. LO 8.1.2

Sally, age 60, has received a $50,000 distribution from $100,000 she is to receive from her ex-husband's (who is age 55) qualified plan account under a qualified domestic relation order (QDRO). Which of the following statements is CORRECT regarding the QDRO and the distributed funds? Irrespective of the plan document, Sally may demand an immediate cash distribution of the remaining funds from the plan trustees. She may roll over the $50,000 distribution into an IRA. Sally's ex-husband is not subject to an early distribution penalty in the execution of the QDRO. Sally may be required to leave the remaining funds with the plan trustee until the earliest time for distributions under the plan. A) II and III B) I, II, and III C) I, II, III, and IV D) II, III, and IV

D) The answer is II, III, and IV. A trustee may not be forced to distribute assets from a plan unless the plan document allows for it. LO 6.5.1

When she retired at age 64, Lauren received a lump-sum distribution from her employer's stock bonus plan. The fair market value of the employer stock contributed to her account was $200,000 at the time of contribution. At the time of the distribution, the employer stock in Lauren's account had a fair market value of $300,000. Six months later, Lauren sold the stock for $310,000. Which of the following statements regarding the sale of Lauren's stock is(are) CORRECT? The $300,000 distribution is taxed at the long-term capital gain rate. Lauren has a $10,000 short-term capital gain when the stock is sold. There was no income tax liability incurred when the stock was contributed to the plan. The net unrealized appreciation (NUA) on the stock is $100,000. A) I and II B) I, II, III, and IV C) IV only D) II, III, and IV

D) The answer is II, III, and IV. Despite being called "the basis" when the stock is withdrawn, there was no income tax liability when the stock was contributed to the plan. Of the $300,000 Lauren received as a lump-sum distribution from the stock bonus plan, $100,000 is net unrealized appreciation (NUA) and will be taxed at the long-term capital gain rate. The remaining $200,000 is taxed at Lauren's ordinary income tax rate in the year of the lump-sum distribution. Because Lauren sold the stock within six months of distribution, the $10,000 post-distribution appreciation is taxed as short-term capital gain. LO 6.1.2

Todd, age 60, has made contributions of $75,000 to his traditional IRA, of which $15,000 were nondeductible contributions. He is considering taking a $20,000 distribution from his IRA, which currently has a fair market value of $175,000. When calculating the nontaxable portion of his IRA, which of the following formulas is CORRECT? Nontaxable portion = nondeductible contributions ÷ [(IRA balance at the beginning of the year + the IRA balance at the end of the year) ÷ 2] × IRA distributions Nontaxable portion = nondeductible contributions ÷ (IRA balance at the beginning of the year + any distributions taken during the year) × IRA distributions Nontaxable portion = [(nondeductible contributions prior to current year + all contributions for current year) ÷ (balances at end of current year + distributions received in current year)] × total distributions during current year A) None of these B) I only C) II only D) III only

D) The answer is III is only. The correct formula is: nontaxable portion = [(nondeductible contributions prior to current year + all contributions for current year) ÷ (balances at end of current year + distributions received in current year)] × total distributions during current year LO 5.3.1

Section 403(b) plan (tax-sheltered annuity plan or TSA) employer contributions A) are based on a maximum annual includible compensation of $265,000 in 2023. B) can discriminate in favor of highly compensated employees. C) are subject to FICA (Social Security and Medicare) and FUTA (federal unemployment) payroll taxes. D) must abide by the annual additions limit.

D) The answer is a Section 403(b) or TSA plan is subject to the annual additions limit of the lesser of 100% of compensation or $66,000 for 2023. Contributions must not discriminate in favor of highly compensated employees. Employer contributions are based on a maximum annual includible compensation of $330,000 and are not subject to FICA and FUTA payroll taxes (employee deferrals are subject to FICA and FUTA). LO 4.3.1

Which of these persons could make tax-deductible contributions to a traditional IRA regardless of their modified adjusted gross income (MAGI)? A) A person who participates in a SEP IRA B) A person who participates in a Section 401(k) plan C) A person who participates in a Section 403(b) plan D) A person who participates in a Section 457 plan

D) The answer is a person who participates in a Section 457 plan. A person who participates in a qualified plan, SEP IRA, or Section 403(b) plan may not be able to make tax-deductible IRA contributions if the participant's MAGI exceeds certain limits. Participation in a Section 457 plan does not subject a person to these limitations. LO 5.1.2

ERISA requirements for qualified plans include A) participation and fiduciary requirements. B) reporting and disclosure. C) coverage and vesting. D) all of these.

D) The answer is all of these. All of these are ERISA requirements for qualified plans. LO 1.1.1

Armor Company has implemented an age-based profit-sharing plan. Under this plan, A) the employer contribution is allocated in such a way that the benefit at retirement is guaranteed. B) compliance with the nondiscrimination (coverage) rules is tested in accordance with contributions rather than benefits. C) younger employees generally receive the greatest allocation. D) allocations to participants are made in proportion to the participant's age-adjusted compensation.

D) The answer is allocations to participants are made in proportion to the participant's age-adjusted compensation. A participant's compensation is age-adjusted by multiplying the participant's actual compensation by a discount factor based on the participant's age and the interest rate elected by the plan sponsor. As a result, older employees generally receive the greatest allocation under an age-based profit-sharing plan. Nondiscrimination rules are tested in accordance with benefits rather than contributions. The final retirement benefit is not guaranteed in any type of profit-sharing plan. LO 3.2.1

All the following statements describe situations in which a target benefit pension plan would best suit the company except A) there is a desire to have a defined benefit plan without the resources to match that desire. B) the workforce has a few older, well-paid owner-employees and key employees compared to the younger and lower paid rank-and-file workers. C) recent economic success and the likelihood of continued success. D) an employee census showing young owners and young rank-and-file employees.

D) The answer is an employee census showing young owners and young rank-and-file employees. A good candidate for a target benefit pension plan is a business that has an employee census showing older owners who are around 50 or older and younger rank-and-file employees. The other choices are additional characteristics that describe good candidates for target benefit pension plans. LO 2.3.2

Which one of these accurately represents the general tax consequences to an employer and employee under a nonqualified plan? A) An employer receives a deduction when asset investments are made to informally fund an employee's nonqualified plan benefit and the employee recognizes an identical amount as income at the same time. B) An employer receives a deduction for a contribution to an employee when paid, and the employee will not be taxed on the contribution until it is withdrawn. C) An employer receives an immediate deduction for a contribution for an employee when paid, and the employee will recognize income when the amount is credited to his or her account is nonforfeitable. D) An employer does not receive a deduction for a contribution to an employee until the employee recognizes the income upon receipt.

D) The answer is an employer does not receive a deduction for a contribution to an employee until the employee recognizes the income upon receipt. An employer receives a deduction for a contribution when an employee recognizes the income, and not before that. LO 9.1.1

Which of these statements regarding a stretch IRA is CORRECT? It allows the IRA owner's beneficiary to name his own beneficiary upon the owner's death. It extends or stretches the period of tax-deferred earnings within an IRA possibly over a decade. A) I only B) II only C) Neither I nor II D) Both I and II

D) The answer is both I and II. A stretch IRA extends or stretches the period of tax-deferred earnings within an IRA beyond the lifetime of the original owner, possibly over a decade. LO 5.2.1

Which of these retirement plans can be integrated with Social Security? A) Employee stock ownership plans (ESOPs) B) Salary reduction SEPs (SARSEPs) C) Savings incentive match plan for employees (SIMPLEs) D) Money purchase pension plans

D) The answer is money purchase plan. Profit-sharing plans, Simplified Employee Pension (SEP) plans, money purchase plans, and defined benefit pension plans may be integrated with Social Security. Employee stock ownership plans (ESOPs), savings incentive match plan for employees (SIMPLEs), and salary reduction SEPs (SARSEPs) are not permitted to use integration. Also, employee elective deferrals and employer matching contributions cannot be integrated. LO 1.3.3

The Department of Labor (DOL) issues A) private-letter rulings. B) guaranty insurance. C) approval of plan documents in the summary plan description. D) rulings, including prohibited transaction exemptions (PTEs).

D) The answer is rulings including prohibited transaction exemptions (PTEs). The Department of Labor issues advisory opinions and rulings (including prohibited transaction exemptions) similar to private-letter rulings, which are issued by the IRS. The DOL does not issue guaranty insurance. The summary plan description is required by the DOL, but the DOL does not approve plan documents. The IRS approves plan documents. LO 1.1.1

A state or local government would choose to establish a Section 457 plan for all the following reasons except A) no early withdrawal penalty on distributions. B) tax-deferred growth of assets. C) the ability of a participant to make elective deferrals. D) tax deductibility of employer contributions.

D) The answer is tax deductibility of employer contributions. Because Section 457 plans are sponsored by tax-exempt entities, deductibility of plan contributions is not an issue and would not be a reason to establish such a plan. A Section 457 plan is not a qualified plan and has no early withdrawal penalty on distributions. LO 4.3.2

Which of these statements is NOT correct regarding the conversion of a traditional IRA to a Roth IRA? A) An amount in a traditional IRA may be transferred in a trustee-to-trustee transfer from the trustee of the traditional IRA to the trustee of the Roth IRA. B) An amount in a traditional IRA may be transferred to a Roth IRA maintained by the same trustee. C) An amount distributed from a traditional IRA can be rolled over to a Roth IRA within 60 days of the distribution. D) The IRA owner's modified adjusted gross income (MAGI) cannot exceed $100,000 in the year of the conversion.

D) The answer is the IRA owner's modified adjusted gross income (MAGI) cannot exceed $100,000 in the year of the conversion. There is no MAGI limit for a taxpayer in the year in which there is a conversion. LO 5.4.2

Larry is a sole proprietor of a business with 15 employees. He would like to implement a formal retirement plan for his business. Larry is 55 years old and is planning to retire in 10 years at age 65. His company currently has a strong cash flow, which is expected to continue. Larry's own personal savings retirement need is $85,000 per year, and he pays himself $95,000 annually. The company can afford to contribute $100,000 this year for Larry's account to any retirement plan that is implemented. Larry will also commit to an annual contribution necessary to fund the retirement plan if needed. Based on limited information, which of the following types of qualified retirement plans would you recommend for Larry and his business? A) Profit-sharing plan B) Stock bonus plan C) Money purchase pension plan D) Traditional defined benefit pension plan

D) The answer is traditional defined benefit pension plan. This type of plan is most appropriate for Larry and his business. The business has favorable cash flow and can commit to the annual contribution required by the defined benefit approach. Additionally, Larry's savings need as a percentage of his compensation exceeds anything possible in a defined contribution plan. Finally, Larry is currently age 55 with only 10 years until retirement. LO 8.1.2

Assume that an employer plans to use corporate-owned life insurance to informally fund a nonqualified deferred compensation agreement and would like to have the flexibility to invest in a number of different asset categories. Which one of these types of life insurance should this employer choose? A) Whole life insurance B) Universal life insurance C) Term insurance D) Variable life insurance

D) The answer is variable life insurance. Variable life insurance permits the cash value of the policy to be invested in a number of different accounts of the insurer, such as indexed equities, blue chip growth, growth and income, international, bonds, and so forth. LO 9.2.1

Required minimum distributions from a qualified plan to a plan participant must be calculated using the Uniform Lifetime Table in all cases except A) when there is no designated beneficiary. B) when there is more than one designated beneficiary. C) when the designated beneficiary is a child under the age of 16. D) when the designated beneficiary is the participant's spouse and the spouse is more than 10 years younger than the participant.

D) The answer is when the designated beneficiary is the participant's spouse and the spouse is more than 10 years younger than the participant. The Uniform Lifetime Table must be used to calculate required minimum distributions under a qualified plan or IRA unless the designated beneficiary is the participant's spouse and the spouse is more than 10 years younger than the participant, in which case the actual joint life expectancy is used. LO 6.4.1


Kaugnay na mga set ng pag-aaral

Chapter 2. The Internet, the Web, and Electronic Commerce

View Set

English Plus 1 second edition Unit 8

View Set